Friday 29 July 2016

What is the limit of this sequence as it approaches infinity

We've got a question that shows us how to find the limit of this sequence as the nth term approaches infinity. I'm unsure of if we should use L'Hopitals rule, or if not what we should use instead. I can see, we may be able to use L'hopital as the formula will be infinity/infinity.



We have that



$$a_n=\frac{n\cos(n\pi+\pi/3)+n(-1)^n}{n^2+1}$$



Then, how evaluate $\lim_{n\to\infty}a_n$?

No comments:

Post a Comment

real analysis - How to find $lim_{hrightarrow 0}frac{sin(ha)}{h}$

How to find $\lim_{h\rightarrow 0}\frac{\sin(ha)}{h}$ without lhopital rule? I know when I use lhopital I easy get $$ \lim_{h\rightarrow 0}...